문제지 PDF 파일을 로드하고 있습니다. 한참 걸릴 수도 있어요 ㅠㅠ 30초 이상 걸리면 새로고침을 한번 해보세요.

2023 7급 국가직 통신이론

1번

  1. 인과성이란 출력이 미래 값에 의존하지 않는다, 즉 현재와 과거 값으로부터만 구성된다는 성질이다. 입력 신호를 x(t)x(t), LTI 시스템의 임펄스 응답을 h(t)h(t), 출력을 y(t)y(t)라고 하면

    y(t)=h(t)x(t)=h(τ)x(tτ)dτ\begin{equation} \begin{split} y(t)&=h(t)*x(t)\\ &=\int_{-\infty}^{\infty}h(\tau)x(t-\tau)d\tau \end{split} \end{equation}

    이다. 현재 시점이 tt이므로 tτ>tτ<0t-\tau\gt t \Leftrightarrow \tau\lt 0이면 이는 미래 시점이다. y(t)y(t)가 이러한 미래 시점의 입력에 의존하지 않음은 h(τ<0)=0h(\tau\lt0)=0이면 보장된다.

  2. 그렇다. 입력과 임펄스 응답과 입력의 콘볼루션이 출력이다. 이는 입력을 임펄스들로 나타내고, 각각의 임펄스들에 대한 응답을 다 더하는 것으로 생각할 수 있다.

  3. 그렇지 않다. 출력도 y(tt0)y(t-t_0)으로 지연된다.

  4. 그렇다. 선형성 때문이다.

따라서 답은 3번 이다.

2번

고등학교에서 배워서 알고 있듯이

Var(X)=E[(XmX)2]=E[X22XmX+mX2]=E[X2]2E[X]mX+mX2=E[X2]2mX2+mX2=E[X2]mX2=E[X2mX2]\begin{equation} \begin{split} Var(X)&=E[(X-m_X)^2]\\ &=E[X^2-2Xm_X+m_X^2]\\ &=E[X^2]-2E[X]m_X+m_X^2\\ &=E[X^2]-2m_X^2+m_X^2\\ &=E[X^2]-m_X^2\\ &=E[X^2-m_X^2] \end{split} \end{equation}

이므로 답은 3번 이다.

3번

확률분포함수란 표현은 처음 본다. 보통은 누적분포함수(cumulative distribution function;cdf)라고 한다. 아무튼

FX(x)=xfX(x~)dx~F_X(x)=\int_{-\infty}^{x}f_X(\tilde{x})d\tilde{x}

이므로

FX(x)={0if x<11x(a(x+1))dx=0x+1axdx=a2(x+1)2if 1x21if x>2F_X(x)= \begin{cases} 0 &\text{if } x\lt -1\\ \int_{-1}^{x}(a(x+1))dx=\int_0^{x+1} axdx=\frac{a}{2}(x+1)^2 &\text{if } -1\le x \le 2\\ 1 &\text{if }x\gt 2 \end{cases}

이다. 식 형태만 봐도 답은 1번 임은 알 수 있긴 하지만 aa를 결정해보자. FX(2)=1F_X(2)=1이어야 하므로

a2(32)=1a=29\begin{equation} \begin{split} \frac{a}{2}(3^2)&=1\\ \Rightarrow a&=\frac{2}{9} \end{split} \end{equation}

이므로 답은 1번이 확실하다.

4번

대역폭은 양수 주파수 범위만을 따진다. 충분히 큰 반송파 주파수로 DSB-SC 변조하게 되면 기저대역에서 음의 주파수 부분도 양수 부분으로 끌려나오게 되므로 양수 범위에서 주파수 스펙트럼은 [W+fc,W+fc][-W+f_c,W+f_c]에 존재한다. 따라서 대역폭은 2W2W이므로 답은 3번 이다.

5번

위상을 미분하면 순시 각주파수가 나오고 이를 2π2\pi로 나누면 순시 주파수가 나온다. 따라서

ϕ(t)=2πfct+20πsin(5t)ω=dϕ(t)dt=2πfc+100πcos(5t)f=ω2π=fc+50cos(5t)\begin{equation} \begin{split} \phi(t)&=2\pi f_c t+20\pi sin(5t)\\ \omega&=\frac{d\phi(t)}{dt}\\ &=2\pi f_c+100\pi cos(5t)\\ f&=\frac{\omega}{2\pi}\\ &=f_c+50cos(5t) \end{split} \end{equation}

이므로 답은 4번 이다.

6번

채널 용량은 입력을 XX, 출력을 YY라 했을 때 둘 사이의 상호 정보량의 최댓값이다. 즉 다음과 같다.

C=maxP[X]I(X;Y)=max(H(Y)H(YX))\begin{equation} \begin{split} C&=max_{P[X]} I(X;Y)\\ &=max(H(Y)-H(Y|X)) \end{split} \end{equation}

이진 대칭 채널에서

H(YX)=xyP(X=x,Y=y)log2P(Y=yX=x)=(p2log2p+p2log2p+1p2log2(1p)+1p2log2(1p))=(plog2p+(1p)log2(1p))\begin{equation} \begin{split} H(Y|X)&=-\sum_x \sum_y P(X=x,Y=y)log_2 P(Y=y|X=x)\\ &=-\left(\frac{p}{2}log_2 p+\frac{p}{2}log_2 p+\frac{1-p}{2}log_2(1-p)+\frac{1-p}{2}log_2(1-p)\right)\\ &=-(plog_2 p+(1-p)log_2(1-p)) \end{split} \end{equation}

이다. 위 값은 P[X]P[X]와는 상관없음과 H(Y)H(Y)P[Y=y]=12P[Y=y]=\frac{1}{2}로 균일할 때 1 bit로 최댓값을 가짐을 고려하면 이진 대칭 채널의 채널 용량은

C=maxP[X](H(Y)+(plog2p+(1p)log2(1p)))=maxP[X]H(Y)+(plog2p+(1p)log2(1p))=1+(plog2p+(1p)log2(1p))\begin{equation} \begin{split} C&=max_{P[X]}\left(H(Y)+(plog_2 p+(1-p)log_2(1-p))\right)\\ &=max_{P[X]}H(Y)+(plog_2 p+(1-p)log_2(1-p))\\ &=1+(plog_2 p+(1-p)log_2(1-p)) \end{split} \end{equation}

이다.

  1. C(1)=1+10+(1p)log2(1p)p1=1+0+0=1C(1)=1+1\cdot 0+(1-p)\cdot log_2(1-p)|_{p\rightarrow 1}=1+0+0=1이다. 1p1-p가 0으로 가는 속도가 log2(1p)log_2(1-p)-\infty로 가는 속도보다 빨라서 그렇다. 로피탈의 정리로 확인해보라.
  2. C(0.5)=1+0.5log20.5+0.5log20.5=1+log20.5=11=0C(0.5)=1+0.5 log_2 0.5+0.5 log_2 0.5=1+log_2 0.5=1-1=0이다.
  3. C(0)=1+plog2pp0+10=1C(0)=1+p\cdot log_2p|_{p\rightarrow 0}+1\cdot 0=1이다. 채널 용량은 확률의 분포 형태에만 관계있기 때문에, 분포 형태가 1번과 같으므로 계산하지 않고도 결과가 1임은 계산하지 않고도 알 수 있다.
  4. 위에서 구한 식과 형태가 다르다.

따라서 답은 1번 이다.

7번

I(X=x1x3x4)=log2121818=log21128=7\begin{equation} \begin{split} I(X=x_1x_3x_4)&=-log_2\frac{1}{2}\frac{1}{8}\frac{1}{8}\\ &=-log_2 \frac{1}{128}\\ &=7 \end{split} \end{equation}

이다. 따라서 답은 4번 이다.

8번

  1. 동기 복조는 당연히 가능하다.
  2. DSB 변조하면 기저대역의 메시지의 주파수 스펙트럼 중 음의 주파수 영역도 양의 주파수 영역으로 끌려나오기 때문에 대역폭이 2배가 된다.
  3. 변조하게 되면 [W+fc,W+fc][-W+f_c,W+f_c][Wfc,Wfc][-W-f_c,W-f_c] 영역에 주파수 스펙트럼이 위치하게 되므로 양측파대가 겹치지 않음은 W+fc>Wfcfc>W-W+f_c\gt W-f_c \Leftrightarrow f_c\gt W임과 동치이다.
  4. 그렇다. 포락선은 양의 부분이기 때문이다.

따라서 답은 2번 이다.

9번

  1. 그렇다. 선형 변조는 AM이다. 이는 각 변조는 삼각함수 안의 값에 메시지가 들어가는데, 삼각함수가 비선형 함수이기 때문이다.
  2. 그렇다. 반송파의 진폭에만 의존한다.
  3. 그렇다.
  4. 그렇지 않다. PM의 경우에는 순시 위상이 2πfct±A2\pi f_c t\pm A이므로 주파수는 fcf_c로 일정하나 위상차이가 있을 것이고, FM의 경우에는 주파수가 fc±Bf_c\pm B 형태로 다를 것이다.

따라서 답은 4번 이다.

10번

최소 전송 속도는

B=30×2×106×log216=60×4×106=240 Mbps\begin{equation} \begin{split} B&=30\times 2\times 10^6 \times log_2 {16}\\ &=60\times 4 \times 10^6\\ &=240\text{ Mbps} \end{split} \end{equation}

이므로 답은 2번 이다.

11번

SSB가 가장 대역폭이 좁다. VSB는 한 측파대는 그대로, 다른 측파대의 일부 잔류이므로 중간이다. 따라서 답은 1번 이다.

12번

  1. E[Y]=E[0.2X0.1]=0.2E[X]0.1E[Y]=E[0.2X-0.1]=0.2E[X]-0.1이다.
  2. E[X+Y]=E[1.2X0.1]=1.2E[X]0.1E[X+Y]=E[1.2X-0.1]=1.2E[X]-0.1이다.
  3. Z=aW+bZ=aW+b이고 WW의 평균을 0, 분산을 σW\sigma_W라고 하자. 분산은 분포의 위치와는 상관없고 분포 형태에만 관련된다. 따라서 b=0b=0으로 놓아도 된다. 그러면
    Var[Z]=E[Z2](E[Z])2=E[Z2]=E[a2W2]=a2E[W2]\begin{equation} \begin{split} Var[Z]&=E[Z^2]-(E[Z])^2\\ &=E[Z^2]\\ &=E[a^2W^2]\\ &=a^2E[W^2] \end{split} \end{equation}
    이다. 즉 어떤 확률변수에 aa를 곱한 확률변수는 상수가 얼마가 여기에 더해지건 원래 확률변수의 분산과 aa의 제곱을 분산으로 갖는다. 따라서 Var[Y]=0.04Var[X]Var[Y]=0.04Var[X]이다.
  4. Var[X+Y]=Var[1.2X0.1]=1.44Var[X]Var[X+Y]=Var[1.2X-0.1]=1.44Var[X]이다.

따라서 답은 2번 이다.

13번

전력 스펙트럼 밀도는 자기상관함수의 푸리에 변환이다. 그리고 출력 전력 스펙트럼 밀도는 입력 전력 스펙트럼 밀도에 H(f)2|H(f)|^2을 곱한 것인데, 주어진 H(f)|H(f)|를 보면 H(f)2=H(f)|H(f)|^2=|H(f)|이다. 따라서 출력의 자기상관함수는 입력의 자기상관함수와 H(f)|H(f)|의 역 푸리에변환인 2f1sinc(2f1τ)2f_1 sinc(2f_1 \tau)의 콘볼루션이다. 한편 입력의 전력 스펙트럼 밀도가 N02\frac{N_0}{2}이므로 입력의 자기상관함수는 N02δ(τ)\frac{N_0}{2}\delta(\tau)이다. 그러므로 출력의 자기상관함수는

R(τ)=2f1sinc(2f1τ)N02δ(τ)=N0f1sinc(2f1τ)\begin{equation} \begin{split} R(\tau)&=2f_1 sinc(2f_1 \tau)*\frac{N_0}{2}\delta(\tau)\\ &=N_0 f_1sinc(2f_1 \tau) \end{split} \end{equation}

이므로 답은 1번 이다.

14번

  1. 그렇다. 여러 신호를 다중화해서 보내는 방법임과 동시에 변조하는 방법이다.
  2. 그렇다.
  3. 그렇다. 부반송파들의 위상이 같은 순간에 진폭이 커지게 된다.
  4. 송신 시 IDFT를 한 다음 CP를 붙인다. 따라서 답은 4번 이다.

15번

n=δ(tnTs)\sum_{n=-\infty}^{\infty}\delta(t-nT_s)를 푸리에 변환해보자. 이 신호는 주기 TsT_s를 가지므로 우선 푸리에 급수로 표현할 수 있다. 푸리에 급수의 계수를 구하면

S[k]=1TsTs2Ts2n=δ(tnTs)ej2πTsktdt=1TsTs2TS2δ(t)ej2πTsktdt=1TsTs2TS2δ(t)ej2πTsk0dt=1TsTs2TS2δ(t)dt=1Ts=fs\begin{equation} \begin{split} S[k]&=\frac{1}{T_s}\int_{-\frac{T_s}{2}}^{\frac{T_s}{2}}\sum_{n=-\infty}^{\infty}\delta(t-nT_s)e^{j\frac{2\pi}{T_s}kt}dt\\ &=\frac{1}{T_s}\int_{-\frac{T_s}{2}}^{\frac{T_S}{2}}\delta(t)e^{j\frac{2\pi}{T_s}kt}dt\\ &=\frac{1}{T_s}\int_{-\frac{T_s}{2}}^{\frac{T_S}{2}}\delta(t)e^{j\frac{2\pi}{T_s}k\cdot 0}dt\\ &=\frac{1}{T_s}\int_{-\frac{T_s}{2}}^{\frac{T_S}{2}}\delta(t)dt\\ &=\frac{1}{T_s}\\ &=f_s \end{split} \end{equation}

따라서

n=δ(tnTs)=k=fsej2πTskt=n=fsej2πfsnt\begin{equation} \begin{split} \sum_{n=-\infty}^{\infty}\delta(t-nT_s)&=\sum_{k=-\infty}^{\infty}f_s e^{j\frac{2\pi}{T_s}kt}\\ =\sum_{n=-\infty}^{\infty}f_s e^{j2\pi f_snt} \end{split} \end{equation}

이다. (마지막에 kknn으로 바꿨다.) 이를 푸리에 변환하면

F{n=fsej2πfsnt}=n=fsej2πfsntej2πftdt=fsn=ej2π(nfsf)tdt=fsn=δ(fnfs)\begin{equation} \begin{split} \mathcal{F} \left\{ \sum_{n=-\infty}^{\infty}f_s e^{j2\pi f_snt} \right\} &= \int_{-\infty}^{\infty}\sum_{n=-\infty}^{\infty}f_s e^{j2\pi f_snt} e^{-j2\pi ft}dt\\ &=f_s\sum_{n=-\infty}^{\infty} \int_{-\infty}^{\infty}e^{j2\pi (nf_s-f)t}dt\\ &=f_s\sum_{n=-\infty}^{\infty} \delta(f-nf_s) \end{split} \end{equation}

이다. 여기서

ej2πftdt=δ(f)\int_{-\infty}^{\infty}e^{j2\pi ft}dt=\delta(f)

를 이용하였다. 이제 gδ(t)g_{\delta}(t)의 푸리에 변환을 구하면

F{gδ(t)}=G(f)fsn=δ(fnfs)=fsn=G(fnfs)\begin{equation} \begin{split} \mathcal{F}\{g_\delta(t)\}&=G(f)*f_s\sum_{n=-\infty}^{\infty} \delta(f-nf_s)\\ &=f_s\sum_{n=-\infty}^{\infty}G(f-nf_s) \end{split} \end{equation}

이므로 답은 3번 이다.

16번

  1. 그렇다. 급수는 주기 신호, 즉 주파수가 주기의 역수인 기본 주파수와 그 배수들로 이루어진 신호에 대한 분석이다.
  2. 그렇다.
  3. 그렇다.
  4. 디리클레 조건은
    f(t)dt<\int_{-\infty}^{\infty}|f(t)|dt\lt\infty
    이다. 바로 위에서 구한 샘플링 신호는 이러한 조건을 만족시키지 않는데도 푸리에 급수를 구할 수 있었다.

따라서 답은 4번 이다.

17번

  1. 스펙트럼 효율은 대역폭 대비 얼마나 많은 신호를 보낼 수 있는가 하는 것, 즉 한 심볼당 얼마나 많은 비트가 실리는가 하는 것이다. MM이 커지면 심볼당 실리는 비트 수가 늘어나므로 스펙트럼 효율은 좋아진다.
  2. 주파수가 아닌 위상 정보이다.
  3. 최소 거리 간격이 멀어진다면 신호 전력이 더 커진다. 따라서 SNR이 높아진다.
  4. 같다.

따라서 답은 3번 이다.

18번

m2(t)m^2(t)의 주파수 스펙트럼은 M(f)M(f)M(f)*M(f)이다. 따라서 이 스펙트럼의 범위를 생각해 보면, 하나의 M(f)M(f)의 양 끝에 M(f)M(f)가 다시 위치하는 형태가 되기 때문에 주파수 스펙트럼의 범위는 2배가 된다. 또는

F{cos2(2πft)}=F{12(cos(4πft)+1)}\mathcal{F}\{cos^2(2\pi ft)\}=\mathcal{F} \left\{\frac{1}{2}(cos(4\pi ft)+1) \right\}

이므로 주파수가 2배가 되는 것을 볼 수 있다. 아무튼 m2(t)m^2(t)의 주파수 스펙트럼은 [2000,2000][-2000,2000]범위에 있기 때문에, 최고 주파수는 2000 Hz이다. 따라서 이의 2배인 4000 Hz로 샘플링해야 하고, 이의 역수는 0.25 msec이니 답은 2번 이다.

19번

설명을 봐도 직관적으로 이해하기 어려운 문제일 것 같다. 고시 합격자한테 물어봤는데 그 형도 잘 납득하진 못했었다고 한다. 먼저 심볼 에러를 생각해보자. kk 비트가 심볼에 실린다고 하면, 심볼에러가 발생하는 경우는 2k12^k-1개가 있다. 이제 이중 어느 한 비트에 에러가 있다고 해보자. 가령 3번째 비트가 원래 1인데 0으로 에러가 발생하는 경우를 생각해보는 것이다. 이는 2k2^k개의 경우 중 3번째 비트가 틀린 경우 절반에 해당한다. 즉 2k12^{k-1}개의 경우가 있다. 따라서 두 경우의 비는

PbPs=2k12k1=M/2M1=47\begin{equation} \begin{split} \frac{P_b}{P_s}&=\frac{2^{k-1}}{2^k-1}\\ &=\frac{M/2}{M-1}\\ &=\frac{4}{7} \end{split} \end{equation}

이다. 그러므로 답은 2번 이다.

20번

주어진 패리티 검사 행렬에서 단위행렬 부분을 제거하면

PT=(101111100111)P^T=\left( \begin{matrix} 1&0&1&1\\ 1&1&1&0\\ 0&1&1&1 \end{matrix} \right)

이다. 따라서

P=(110011111101)P=\left( \begin{matrix} 1&1&0\\ 0&1&1\\ 1&1&1\\ 1&0&1 \end{matrix} \right)

이다. 그러므로 생성행렬은

G=(PI)=(1101000011010011100101010001)\begin{equation} \begin{split} G&=\left( \begin{matrix} P&I \end{matrix} \right)\\ &=\left( \begin{matrix} 1&1&0&1&0&0&0\\ 0&1&1&0&1&0&0\\ 1&1&1&0&0&1&0\\ 1&0&1&0&0&0&1 \end{matrix} \right) \end{split} \end{equation}

이를 이용해서 부호어를 구해보면

c=MG=(1001)(1101000011010011100101010001)=(0111001)\begin{equation} \begin{split} c&=MG\\ &=(\begin{matrix} 1&0&0&1 \end{matrix} )\left(\begin{matrix} 1&1&0&1&0&0&0\\ 0&1&1&0&1&0&0\\ 1&1&1&0&0&1&0\\ 1&0&1&0&0&0&1 \end{matrix} \right)\\ &=(\begin{matrix} 0&1&1&1&0&0&1 \end{matrix}) \end{split} \end{equation}

이므로 답은 3번 이다.

21번

H(f)=0.5ej2πf×12H(f)=0.5e^{-j2\pi f\times\frac{1}{2}}

이다. 따라서 이러한 시스템의 출력은 입력 대비 시간 지연이 12\frac{1}{2}만큼 일어나고 크기는 절반이 된다. 그러므로 답은 2번 이다.

22번

  1. 그렇지 않다. 심볼 에너지가 동일하면 QPSK의 비트당 에너지는 BPSK의 절반이다. 즉 서로 다른 비트 심볼과의 거리가 절반이 되므로 오류확률이 더 크다. I채널과 Q채널이 독립이라 비트오류확률이 같다는 명제는 비트당 에너지가 동일할 때 성립한다.
  2. 그렇다. 심볼당 실리는 비트가 두 배이기 때문이다.
  3. 그렇다.
  4. 그렇다.

따라서 답은 1번 이다.

23번

  1. 모두 -1인 수열은 제외해야 하므로 2m12^m-1의 길이(주기)를 갖는다.
  2. 한 주기 내에서 이 떄에는, 그리고 이 때에만 코드의 자기상관이 최대가 된다. 1이든 -1이든 다 제곱되어서 더해지기 때문이다. 이외의 경우에는 똑같이 자기상관이 -1이다.
  3. 그렇다. 이를 Balance Property라고 한다.
  4. 나타날 수 있다. 안 그러면 코드는 1로 시작하는 거 하나, -1로 시작하는 거 하나 해서 총 두 개밖에 없을 것이다. 오히려 Run Property에 의해 연속으로 나타나는 규칙이 있다.

따라서 답은 3번 이다.

24번

  1. x(t)x(t)가 실수 신호라면 Xn=1Ts0Tsx(t)ej2πTsntdtX_n=\frac{1}{T_s}\int_0^{T_s}x(t)e^{-j\frac{2\pi}{T_s}nt}dt, Xn=1Ts0Tsx(t)e+j2πTsntdt=XnX_{-n}=\frac{1}{T_s}\int_0^{T_s}x(t)e^{+j\frac{2\pi}{T_s}nt}dt=X_n^*관계를 만족한다. 주어진 조건은 이를 만족하므로 x(t)x(t)는 실수 신호이다.
  2. 기본 주파수는 주기의 역수이므로 5 Hz이다.
  3. X3=jπX_3=-\frac{j}{\pi}이므로 이 주파수의 진폭 스펙트럼 성분은 1π\frac{1}{\pi}이다.
  4. X5=3j5πX_5=-\frac{3j}{5\pi}이므로 이 주파수의 위상 스펙트럼 성분은 π2-\frac{\pi}{2}이다.

따라서 답은 4번 이다.

25번

복호에러확률은 전체 확률에서 모두 맞을 확률과 하나가 틀릴 확률을 뺀 것이다. 따라서

Pe=10.833×0.2×0.82=1641253×15×1625=164+48125=13125\begin{equation} \begin{split} P_e&=1-0.8^3-3\times0.2\times0.8^2\\ &=1-\frac{64}{125}-3\times\frac{1}{5}\times\frac{16}{25}\\ &=1-\frac{64+48}{125}\\ &=\frac{13}{125} \end{split} \end{equation}

이므로 답은 1번 이다.